Вы находитесь на странице: 1из 40

Florida Board of Professional Engineers

2507 Callaway Road


Suite 200
Tallahassee, Florida 32303

Study Guide

1
INTRODUCTION

The Florida Board of Professional Engineers is responsible for enforcing Chapter 471 of the
Florida Statutes, which states that the Legislature deems it necessary in the interest of public
health and safety to regulate the practice of engineering in this state.

In responding to this statutory mandate, the Florida Board of Professional Engineers has made
marked strides to ensure that licensees are competent and in fact do not pose a danger to the
public. One aspect of implementing the law emphasizes specific requirements in the evaluation
of a candidates ability and knowledge of professional engineering. This evaluation is currently
accomplished through the use of the national examination developed by the National Council of
Examiners for Engineering and Surveying (NCEES). The second means of insuring a persons
ability to practice in Florida is through completion of this Study Guide on Florida's Laws and
Rules. The Study Guide consists of a written text that thoroughly reviews Chapter 471, Florida
Statutes and Chapter 61G15, Florida Administrative Code.

You may access the Study Guide by going to www.studyguide.com and putting in your email
address and the password provided in the receipt of application letter. You may print out the
Study Guide and Study Guide questions; however, the Study Guide is to be completed online.
You may stay online as long as you want or you may go on and off line as frequently as you
want; however, once you answer the last question the quiz will be graded and your score sent
directly to your file at the FBPE office.

You are required to submit the Study Guide as a part of the application process. A file will not
be considered complete unless the Study Guide is submitted. In accordance with Rule 61G15-
20.0016, F.A.C., an applicant must achieve a minimum score of 90% on the Study Guide. For
the individuals who are applying with foreign degrees, this Study Guide is recognized as
satisfying the Professionalism and Ethics course requirement.

In addition, this text may serve as an excellent resource for information on the Engineer Practice Act and
the rules promulgated by the Florida Board of Professional Engineers, as well as links to numerous web
sites containing important information for engineers.

Upon completion of the Study Guide, the applicant will have an understanding of:
The history of the Florida Board of Professional Engineers, the requirements for service on the
Board and the current membership
The history of the Florida Engineers Management Corporation
The Law and Rules that are the basis for regulation of the profession and definitions that are key
to their understanding
The licensure of engineers to include who is exempt from licensure, pathways to licensure, and
the requirements of the different pathways
The requirements to maintain the license and the various states of licensure
Requirements relating to sealing, signing and dating engineering documents, as well as the proper
process for adopting the work of another engineer as that of your own
The Responsibility Rules, as well as definitions key to their understanding
The disciplinary process when the law and rules are violated, including a review of the type acts
that constitute grounds for disciplinary action and penalties that may be imposed

2
CHAPTER 1
General Information

About the FBPE


It is of interest to know that The Florida Engineers Practice Act was first enacted by the Florida
Legislature in May, 1917, and was the first all inclusive Engineer Practice Act passed in the United
States. The first Board was appointed on July 17, 1917, and began functioning September 11, 1917.
Those very first Board members were:
R.E. Chandler, Chairman
Orrin Randolph, Vice Chairman
C.S. Hammatt, Secretary Treasurer
R.Y. Patterson
Gail L Barnard
Obviously, you wont see any of those names among the current members of the Board;. However, you
will see C.S. Hammatts name as an example as how a seal should read in 61G15-23.001, F.A.C. Seals
Acceptable to the Board.

In 1925, the law was amended to require that an applicant be subject to an examination. In 1941, the
chapter was substantially revised and re-enacted and designated as Chapter 471, F.S., the number it has
held since. In 1963, the portion of the law allowing corporations and partnerships to offer engineering
services was added and two years later the number of Board members was increased to seven. Also in
1965 the Board was authorized to adopt Rules of Professional Conduct, the precursor of todays Chapter
61G15, F.A.C., Rules of the Board of Professional Engineers, and made them binding on all who held a
Certificate of Registration, the precursor of todays License.

In 1979, during a periodic Sunset Review of Chapter 471, F.S., the legislature separated the regulation
of the practice of engineering from that of land surveying and recreated the Board of Professional
Engineers with seven engineering members and two public members, or persons with no ties to the
engineering profession. Those in the profession of Land Surveying, as it was known then, were provided
a Board of Professional Land Surveyors which, subsequently, morphed into what is now the Board of
Land Surveyors and Mappers.

In 1998, the Florida Engineers Management Corporation was created to provide administrative,
investigative and prosecutorial services to the Board and in 2004 the number of Board members was
increased to eleven.

Requirements for membership on the FBPE


Chapter 471.007, F.S., sets forth the makeup of the Board as follows
Three shall be civil engineers
One shall be a structural engineer
One shall be either an electrical or electronic engineer
One shall be a mechanical engineer
One shall be an industrial engineer
One shall be an engineer educator
One shall be from any discipline of engineering other than civil engineering
Two shall be laypersons who are not and have never been engineers or any closely related
profession or occupation

3
It is odd that the law calls for separation of the profession by discipline; however, the law does not license
engineers by discipline. In Florida, licensure is as a Professional Engineer and persons may practice
within any discipline in which they are competent by virtue of their education, training and experience.
Members are named by the governor and subject to confirmation by the Senate. They serve a term of four
years. A person can be appointed for a second four-year term.

The importance of good appointments to the board cannot be overstated. Unlike many gubernatorial
appointments, an appointment to the Board of Professional Engineers is not ceremonial. The Board of
Professional Engineers is a working board and the importance and amount of work is substantial. The
board meets, on the average, every other month for two full days, and many times those full board
meetings are preceded by board committee meetings of a day or more. Members of the board are paid $50
a day plus per diem and travel expenses r. By the way, they do not get paid while at home performing the
necessary review of voluminous materials that flow from each board or committee meeting. The currency
for remuneration for board service is honor and it is certainly a high honor to be named to a board
responsible for insuring that the publics health, safety and welfare is guarded from less than capable or
charlatan engineers or unlicensed persons.

Who is on the FBPE


On the date this Study Guide was written, the following named persons were serving on the Board.
Obviously, since this writing, terms may have expired and new members may have been appointed. For
an up-to-date list, you may visit www.fbpe.org and click on fbpe.
John Burke, P.E., Chair (Electrical)
H. Dann Wallis, P.E., Vice Chair (Industrial)
David O. Charland, P.E.,(Structural)
R. Scott Batterson, P.E. (Civil)
Bijay Panigrahi, Ph.D., P.E. (Civil)
Christian Bauer, Ph.D., P.E. (Industrial)
Warren Hahn, P.E., (Mechanical)
Jonathan Earle, Ph.D., P.E. (Education)
Nola A. Garcia de Quevedo (Public Member)
Mary Young, (Public Member)
(One Civil position vacant)

Where is the headquarters?


Chapter 471.009, F.S., is very short and very specific. It requires the location of the Board of
Professional Engineers to be in Leon County. It is physically located at 2507 Callaway Road, Suite 200,
Tallahassee, Florida 32303-5268. The phone number is 850-521-0500 and the fax is 850-521-0521. As
previously mentioned, the web site is www.fbpe.org.

If the acronym for the Florida Board of Professional Engineers is FBPE, what does FEMC
stand for?
FEMC is the acronym for the Florida Engineers Management Corporation and is a statutorily created,
non-profit corporation whose mission is to deliver administrative, investigative and prosecutorial services
to the Board of Professional Engineers. In other words, FEMC is the staff of the board and is responsible
for all aspects of administration, from answering the telephones to investigating complaints to prosecuting
violations of the Engineer Practice Act. It is a unique service-delivery system created by the 1998
legislature to serve the FBPE and improve protection to the public by pin pointing staff support
responsibilities and reducing bureaucracy. FEMC is required to meet certain performance standards and
those are found in 61G15-37.001 F.A.C. (the very last rule in 61G15). The performance standards are not

4
inordinately difficult and not much different from what can be expected of any state agency. Several,
however, should be noted in as much as they require FEMC to:
o Send a reminder notice to each licensee at least 90 days before the end of the licensing cycle.
The heads up here is that it will go to the last known address FEMC has for you, so if you
have moved and not notified FEMC, the renewal notice will go to the old address and, in all
likelihood, will not be received. The suggestion is, once you have become licensed go to
https://www.myfloridalicense.com/licensing and check your address of record. If the address
is not correct, go to http://www.fbpe.org and click on applications and look for the
Address Change Form, which can be completed on line if you have Adobe 7.0. Otherwise,
download the form and fax it to the FEMC office at 850-521-0521. By the way, unlike many
businesses, there is no second or final reminder. If you fail to respond, your license will be
delinquent on March 1 (the first day of the licensure period) and the next reminder you
receive will be 21 months later or 90 days prior to your license being null and void.
o Respond to an applicant within 30 days of receipt of an application for licensure and notify
the applicant of any errors or omissions in the application. Generally, contacting an applicant
about missing or needed information within 30 days isnt an issue. However, twice a year, a
thousand or so applicants wait until the deadline to submit their applications and the process
may bog down. This is another way of saying the standard is good and in place, but human
nature may make it difficult to meet.
o Make a determination of legal sufficiency within 30 days of receiving a complaint and
provide the subject of the complaint or his or her attorney with a copy of the complaint within
15 days of determining the complaint to be sufficient. More on this subject under the
Disciplining section, but for now, understand that this is a reasonably tough standard and
rarely met in pre-FEMC conditions.

Who provides services to the FBPE?


The FBPE is considered a working board in that members of the FBPE are heavily involved in board
duties and make key policy decisions. However, the day-to-day work of the FBPE is done by employees
of the Florida Engineers Management Corporation usually referred to as board staff.

The email address of each of the board staff, the DBPR Contract Administrator and the Board Counsel
can be found on the board web site at www.fbpe.org under the FBPE tab (scroll down past the FBPE
members). Keep in mind there are over 30,000 licensed engineers and 3,000 plus certificates of
authorization holders, all of whom seek to renew their license or certificate of authorization in the 90 day
or so period between receipt of the renewal notice and the February 28 deadline for renewal. Factor in the
several hundred who have applied to become licensed and the number of engineers whose license is under
investigation and you can see that the Board office is a busy place. Although many calls or emails to the
Board staff are routine, many are complex and directly involve the professional livelihood of engineers.
So be patient and understand that protecting the public from less than adequate engineering is important
and, sometimes, time consuming. You will want the same deliberate effort when you ask your question.

Chapter 471 F.S., The Engineer Practice Act


Chapter 471 of the Florida Statutes (F.S. for short), relates only to the engineering profession. The
Rules of the board (Chapter 61G15 Florida Administrative Code or F.A.C.) do not expand the authority
of the board beyond the authority provided by Chapter 471 F.S.., but provide more details and mechanics.
Chapter 471 F.S. and Chapter 61G15 F.A.C. will be covered in depth in subsequent chapters of this guide.
However, before moving into the details, it is important that we come to an understanding of certain terms
and their legal definition.

5
You can download up-to-date copies of Chapter 471 F.S. (The Florida Engineer Practice Act) and 61G15
F.A.C. (The Rules of the Board) from www.fbpe.org., which is the Florida Board of Professional
Engineers web site.

Definitions Important to Engineers


Chapter 471.005 F.S. provides key definitions, all of which are important to understanding the rules and
regulations of the engineering profession.
471.005(1) defines Board as Board of Professional Engineers
471.005(2) defines Board of Directors as the Board of Directors of the Florida Management
Corporation (the company that provides services to the FBPE)
471.005(3) defines Certificate of Authorization as a license to practice engineering by a
corporation or partnership
471.005(4) defines Department as the Department of Business and Professional Regulation
471.005(5) defines Engineer to include the term professional engineer and licensed engineer,
and means a person who is licensed to engage in the practice of engineering
471.005(6) defines Engineer Intern as person who has graduated from an approved engineering
curriculum and passed the fundamentals of engineering exam. Note the term replaces the now
dated term engineer-in-training.
471.005(7) defines Engineering and, as it is the most important term in the chapter, it is printed
in full below, followed by a commentary on the more significant provisions in the definition:

(7) Engineering includes the term professional engineering and means any service or creative
work, the adequate performance of which requires engineering education, training, and
experience in the application of special knowledge of the mathematical, physical, and engineering
sciences to such services or creative work as consultation, investigation, evaluation, planning
and design of engineering works and systems, planning the use of land and water, teaching of the
principles and methods of engineering design, engineering surveys, and the inspection of
construction for the purpose of determining in general if the work is proceeding in compliance
with drawings and specifications, any of which embraces such services or work, either public or
private, in connection with any utilities, structures, buildings, machines, equipment, processes,
work systems, projects, and industrial or consumer products or equipment of a mechanical,
electrical, hydraulic, pneumatic, or thermal nature, insofar as they involve safeguarding life,
health, or property; and includes such other professional services as may be necessary to the
planning, progress and completion of any engineering services. A person who practices any
branch of engineering; who by verbal claim, sign, advertisement, letterhead, or card, or in any
other way, represents himself or herself to be an engineer or, through the use of some other title,
implies that he or she is an engineer or that he or she is licensed under this chapter; or who holds
himself or herself out as able to perform, or does perform, any engineering service or work or any
other service designated by the practitioner which is recognized as engineering shall be construed
to practice or offer to practice engineering within the meaning and intent of this chapter.

This definition is the essence of the Engineer Practice Act and contains terms important to the
admission to the practice and the practice of engineering. As the definition is importantand
lengthy, we have broken the definition into bite size portions so that it can be better understood and
recalled as other portions of the Engineer Practice Act are covered.

Engineering includes the term professional engineering and means


any service or creative work, the adequate performance of which requires engineering
education, training and experience

6
in the application of special knowledge of the mathematical, physical and engineering
sciences
to such services or creative work as
o consultation
o investigation
o evaluation
o planning
o design of engineering systems
o planning the use of land and water
o teaching the principles and methods of engineering design
o engineering surveys
o and the inspection of construction for the purpose of determining in general if the
work is proceeding in compliance with drawings and specifications
any of which embraces such services or work, either public or private, in connection with
any
o utilities
o structures
o buildings
o machines
o equipment
o processes
o work systems
o projects
o industrial or consumer products
o equipment of a mechanical, electrical, hydraulic, pneumatic or thermal nature
...insofar as they involve safe guarding life, health or property, and
includes such professional services as may be necessary to the planning, progress and
completion of any engineering services.

A second sentence in the definition further expands the definition of engineering and its application
by saying
A person who practices any branch of engineering who by
o verbal claim
o sign
o advertisement
o letterhead
o card
o or in any other way
represents himself or herself to be an engineer
or through the use of some other title implies that
o he or she is an engineer or
o he or she is licensed under this chapter or
o who holds himself or herself out as able to perform or
o does perform
o any engineering service or work or
o any other service designated by the practitioner which is recognized as engineering
shall be construed to practice or offer to practice engineering within the meaning and intent
of this chapter.

7
471.005(8) defines license as the licensing of engineers or the certification of a business to
practice engineering
471.005(10) defines a retired professional engineer or professional engineer, retired as one
who
o has been licensed and
o chooses to relinquish or not renew his license
o is approved by the board to be granted the title Professional Engineer, Retired
Those who may consider the retired professional engineer status need to know that should you
change your mind after the status change is granted, you will be required to meet licensure
standards as they exist when you re-apply.
471.005(11) says that the term secretary means the Secretary of the Department of Business
and Professional Regulation

8
CHAPTER 2
Licensure

Who Is Exempt from Licensure?


As strange as it seems, the authors of Chapter 471, the Engineer Practice Act, do not deal with who is
required to be licensed until those exempt from licensure under the Engineer Practice Act are spelled out.
That is done in Chapter 471.003, F.S.
., where a substantial list of exemptions appears. Although most readers of this Study Guide desire
licensure as engineers, it may be helpful for them to know who doesnt need to be licensed.

Included in the exemptions in 471.003 F.S. are:


(2)(a) Any person practicing engineering on property owned by her or him unless the practice
involves a public utility or public health, safety, or welfare or the safety and welfare of
employees.
(2)(b)1 Persons acting as a public officer employed by a government working on projects of less
than $10,000.
(2)(b) 2 Employees of government who are subordinates under a person who is licensed and in
responsible charge, as long certain supervision standards are met.
(2)(c) Full-time employees of corporations whose practice is limited to the design or fabrication
of manufactured products and servicing such products. (a.k.a. the industry exemption)
(2)(d) Full-time employees of a public utility
(2)(e) Full-time employees who are subordinates of a person in responsible charge who is
licensed
(2)(f) Contractors in the execution of work done by a professional engineer
(2)(g) A surveyor who contracts for engineering services incidental to his surveying practice and
who delegates the engineering services to a qualified licensed engineer either in his firm or under
contract with his firm
(20(h) Any electrical, plumbing, air conditioning or mechanical contractor whose practice
includes the design and fabrication of electrical, plumbing, air conditioning or mechanical
systems, which he installs by virtue of a license under chapter 489 F.S. (The Construction
Industry Licensing Law) provided:
o The electrical or plumbing or HVAC system has a value of less than $50,000
o The aggregate service is less than 600 amperes (240 volts) on residential systems or less
than 800 amperes (240 volts) on commercial or industrial electrical systems
o The plumbing system has less than 250 fixture units
o The HVAC system does not exceed a 15 ton-per-system capacity or is designed for less
than a 100-person capacity
(2)(i) A licensed contractor when negotiating under a design build contract, as long as the
engineering services provided under the contract are performed by a licensed engineer
(2)(j) Any aerospace company or anyone working for an aerospace company who provides
engineering for any space-related aircraft or launch vehicle

The incidental practice exemption

9
One exemption from the Engineering Practice Act, 471.003(3), has been controversial for many years and
may remain so into the foreseeable future. It is called the incidental practice provision and is the
product of long discussions and compromise between the architectural and engineering community. The
exact same language appears in both the Engineering Practice Act and the architects practice act. The
exact language appears below but, in essence, it means a civil engineer, or persons in his responsible
charge, can perform architectural services that are merely incidental to the engineering aspects of the
project. Likewise, an architect can perform services considered to be engineering, and so long as those
services are incidental to the architects practice of architecture, the architect is exempt from licensure
under Chapter 471 F.S..

Sowhat is the definition of incidental? Down through the years, the FBPE has adopted a position
much like the response of a judge when asked the definition of pornography which was, Ill know it
when I see it. Currently, neither the FBPE nor the Board of Architecture and Interior Design have been
pressed to make such a call. However, in times of slower construction, the issue usually arose when
architects would file an unlicensed practice complaint against an engineer for designing a building. The
position of the complaining architect was that only an architect could take design responsibility for a
building. The counter argument by the engineer was that buildings were included in the definition of
engineering, and that the test for a proper submission of building plans was not that the plans were
sealed by an architect or an engineer, but whether the plans that were submitted conformed to all
applicable building codes.

All that having been said, architects have an exemption from the Engineer Practice Act as long as the
engineering services provided are incidental to the services they provide as an architect. By the way, as
the wording of the statute below states, in no instance may an engineer use the term architect and no
architect may use the term engineer.

471.003(3) reads in full Notwithstanding the provisions of this chapter or any other law, no licensed
engineer whose principle practice is civil or structural engineering, or employee or subordinate under
responsible supervision or control of the engineer, is precluded from performing architectural services
which are purely incidental to her or his engineering practice, nor is any licensed architect, or employee
or subordinate under responsible supervision or control of the architect, precluded from performing
engineering services which are purely incidental to her or his architectural practice. However, no
engineer shall practice architecture or use the designation architect or any term derived there from, and
no architect shall practice engineering or use the designation engineer or any term derived there from.

Faculty Exemption
The last exemption in the Engineer Practice Act was placed there at the request of engineering deans and
faculty of the state university system, as they felt the requirement for licensure as a professional engineer
was an impediment to recruiting engineering faculty. The teaching of the principles and methods of
engineering design remains in the definition of engineering to allow those who teach to count the time
teaching toward the four years of engineering experience. However, 471.0035 F.S., exempts persons
employed by post secondary educational institutions from licensure as a professional engineer.

Who needs to be licensed?


Recall earlier in the Study Guide the couple of pages dealing with the definition of engineering?
Persons doing those activities as defined in Chapter 471.005(7) F.S. and not exempt under Chapter
471.003 F.S. are required to be licensed.

Who can become licensed?


Having stated who does not need to become licensed as a professional engineer and, understanding all
others performing those services defined as engineering are required to be licensed, then what are the

10
requirements for licensure and how does one become licensed? Chapters 471.013 and 471.015 provide
this information.

Lets discuss the second question first. There are several ways to become licensed, all of which include
the blanket requirement that the person be of good moral character:

By Examination
Licensure by examination means obtaining a score of 70 or better on the National Council of
Examiners for Engineering and Surveying (NCEES) fundamentals exam and the principles and
practice exam and providing proof of four years active engineering experience of a character
indicating competence to be in responsible charge provided, however, the applicant is:
A graduate of an approved engineering curriculum of four years or more in a school, college or
university approved by the FBPE, or;
A graduate of an engineering technology curriculum of four years or more in a school in the state
university system, as long as he or she had enrolled and graduated prior to July 1, 1979, or;
In lieu of such education and experience requirements he or she had 10 years or more of active
engineering work of a character indicating the applicant is competent to be placed in responsible
charge of engineering. This provision cannot be utilized unless the applicant notified the
Department of Business and Professional Regulation of the intent to apply under this provision
before July 1, 1984, and the applicant was engaged in such work on July 1, 1981.

Practically speaking, the clock has run out for applicants with a technology degree or ten years of
experience, so the vast majority of applicants today are graduates of engineering programs accredited by
the Accreditation Board for Engineering and Technology (ABET). Under Chapter 61G15-10.006, F.A.C.,
the FBPE can accept graduates from schools other than those with ABET approved curricula. However,
an evaluation of the applicants transcript must determine if the program and transcript are comparable to
an ABET program.

Foreign Degree Applicants


As ABET does not involve itself with foreign degree programs, applicants with degrees from foreign
institutions are required to document substantial equivalency to ABET criteria. Chapter 61G15-
20.007(2)(a) through (2)(d) sets forth the criteria that must be met by a foreign student and, in summary,
includes:
32 college credit hours of higher mathematics and basic sciences
16 college credit hours in humanities and social sciences
48 college credit hours in engineering sciences and engineering design
Evidence of appropriate laboratory experience, computer skill and understanding of ethical,
social, economic and safety considerations of the engineering practice, as well as competency in
the English language as evidenced by a minimum score of 550 on the written and 213 on the
computer based Test of English as a Foreign Language or TOEFL exam.
The applicant is responsible for getting the evaluation of substantial equivalency from a provider
of the service approved by the FBPE. The FBPE Educational Advisory Committee will make the
final decision regarding equivalency and will recommend to the FBPE whether or not the
applicant will be approved for admittance to the exam or by endorsement.

The Exams
An applicant may be admitted to the required fundamentals exam during the final year of undergraduate
study, or as a graduate of an approved engineering curriculum in a school or university approved by the

11
FBPE. An applicant who has a Ph.D. in engineering from an institution that has an undergraduate
program approved by the Engineering Accreditation Commission of the Accreditation Board for
Engineering Technology, and who has taught at least three years at the baccalaureate level after receiving
the Ph.D., is deemed to have passed the fundamentals exam. Applicants must successfully complete the
fundamentals exam before sitting for the principles and practice exam.

If an applicant fails the fundamental or the principles and practice exam three times the FBPE must
require the applicant to take 12 hours of additional college level classes with grades no lower than a C
or its equivalent as a condition of future eligibility to take the exam.

By Endorsement

Licensure by endorsement means the applicant is qualified to and has taken the NCEES fundamentals
exam and the principles and practice exam or their equivalents, and has completed the requisite four years
engineering experience, or holds a license in another state if the criteria for issuance of the license was
substantially equivalent as that existing in Florida at the time the license was issued.

Most licenses issued by endorsement are issued on the basis of the applicant having a license in another
state, with the issuing requirements of that state being the same or more stringent than the requirements of
Florida at the time of issuance. In fact, the FBPE has instituted the National Council of Examiners for
Engineering and Surveying Model Law Engineer fast-track licensing process for applicants who are
licensed in another state, meet the ABET degree requirements and have the four years of needed
experience by allowing the license to be issued by the FBPE staff without the needed approval of the full
board.

The Engineering Practice Act deems equivalent to having passed the fundamentals exam an applicant
who has:
Had a license in another state for 15 years and has had 20 years of continuous professional-level
engineering experience;
Received a doctorate degree in engineering from an institution that has an undergraduate
engineering program accredited by the Accreditation Board for Engineering Technology (ABET);
or
Received a doctorate degree in engineering and has taught engineering full time for at least three
years at the baccalaureate level or higher after receiving the degree.

The Engineering Practice Act deems equivalent to having passed the fundamentals and the principles
and practice exams an applicant who has held a license in another state for 25 years and has 30 years of
continuous professional-level engineering experience.
If an applicant applying for licensure by endorsement has taken either the fundamentals or the principles
and practice exam more than five times, he must document that he has taken the needed remedial
academic courses, as would any applicant applying for licensure. An applicant for licensure under
endorsement whose only deficiency involves humanities and social sciences is deemed to have satisfied
the requirement if he or she has held a license and practiced in another jurisdiction for two or more years.

With continuing education requirements now becoming popular in many states, some engineers who hold
licenses in many jurisdictions are allowing licenses they may hold in a state in which they infrequently do
work to lapse, thinking they may be able to avoid a renewal fee and the costs of meeting continuing
education requirements by merely re-applying by endorsement should they need an active license. Be
aware that Florida law requires that an applicant who previously held a license in Florida but allowed the
license to become null and void, however, they will be required to apply by endorsement and the

12
applicant must meet the licensure requirements that exist at the time of application. This may not be
significant for the Model Law Engineer. However, for someone licensed under the 10 year experience
provision, or by graduating from a engineering technology program instead of an ABET engineering
program, licensure by endorsement may be problematic.

Four Years Experience


Whether seeking licensure by exam or by endorsement, an applicant must have four years of acceptable
experience in engineering at the time the application is submitted. To be acceptable, the type of
experience must involve activities in the field of engineering as defined in Chapter 471.005(7). Chapter
61G15-20.002 contains guidelines the FBPE follows when determining applicable experience. These
guidelines are:
The experience should logically follow and constitute an application of engineering experience
previously obtained.
As engineering experience obtained prior to graduation is usually sub-professional in nature, it
may be awarded based on 25% of actual time. If the experience is gained after a substantial
number of engineering design courses and is of moderate complexity, the experience may be
awarded up to 50% of actual time. In no instance can pre-graduation experience count more than
12 months.
Experience is based on a 40-hour week and no credit is given for overtime work
No credit is given for part-time work while pursuing engineering education on a full time basis.
No credit will be given for part time pursuit of a masters or doctorate while obtaining full-time
work experience.
Experience must be progressive and indicate increasing responsibility.
Experience must not be obtained in violation of the Engineer Practice Act.
Experience gained in the armed services must be of a character equivalent to that which would
have been gained in the civilian sector.
Experience should be gained under the supervision of a licensed professional engineer. If not, an
explanation as to why it should be accepted is needed.
For experience in sales to be credited, it must be shown that engineering principles were required
and used in gaining the experience.
For teaching experience to be creditable, it must be in engineering courses at an advanced level in
an institution approved by the FBPE.
Experience can be gained in engineering research and design projects by members of an
engineering faculty where the program is approved by the FBPE.
Experience should include a demonstration of knowledge of engineering mathematics, physical
and applied science, properties of materials and fundamentals of principles of engineering design.
Experience should include a demonstration of the application of engineering principles in the
practical solutions to an engineering problem.
The FBPE will accept as one year experience a masters degree from an FBPE- approved
institution.
The FBPE will accept as one year experience a doctorate degree from an FBPE- approved
institution.

In order to verify an applicants experience, the FBPE will require evidence of employment from
employers or supervisors who are employed in the engineering profession, or who are professional
engineers and able to state the quality and character of the applicants duties and responsibilities. In
addition to employer verification, the applicant must list three personal references who are professional
engineers. If the information submitted is insufficient or incomplete, the applicant will be required to
supplement the information so that an intelligent decision may be made on whether admittance to the
examination is allowable.

13
Good Moral Character
In addition to a proper education, two days of testing and four years of applicable experience, there is also
a requirement for good moral character. Fortunately (or unfortunately) there is no definition for what
good moral character may be. However, the FBPE may refuse to certify an applicant for licensure for
failure to satisfy the good moral character requirement if there is a connection between the lack of good
moral character and the responsibilities of a professional engineer. The finding of the lack of good moral
character must be supported by clear and convincing evidence. When an applicant is found unqualified
based on this provision, he must be provided the complete record of evidence and notice of his rights to a
hearing and appeal.

Maintaining the License


Upon receiving a professional engineer license, there are certain requirements that must be met in order
for it to be renewed at the end of the licensure period. Chapter 471.017 deals with renewal of licenses and
charges the Florida Engineers Management Corporation to renew licenses upon the receipt of a fee and an
application. This section also requires the FBPE to adopt rules establishing a procedure for the biennial
renewal process. As a part of the rules, the statute requires an applicant for renewal to demonstrate
continuing professional competency by completing a total of eight professional development hours (PDH
s) during the biennium. Although the statute directs the rules to be consistent with the guidelines of the
National Council of Examiners for Engineers and Surveyors (NCEES), which discourages continuing-
education requirements that are particular to one state as it impacts the mobility of a license, the same
paragraph dictates that four of the professional development hours must be on Chapter 471 F.S., the
Engineer Practice Act and the Rules of the Board of Professional Engineers (Chapter 61G15, F.A.C.).
The remaining four hours must relate to the engineers area of practice.

The rules the statute directs the FBPE to make regarding continuing education are found in 61G15-22
F.A.C. These rules require the engineer to certify that the eight hours of continuing education have been
completed. The rules define area of practice as an engineering discipline for which the principles and
practice exam is offered by NCEES. There 18 such exams and they can be found on the NCEES web site
at http://www.ncees.org/exams/professional/. The Professional Development Hour is defined as 50
minutes and is stated to be the common denominator for other units of credits. Continuing Education
Units are recognized as a customarily used unit of credit and one CEU is equal to 10 hours of class, or 10
PDHs.

61G15-22.002 F.A.C., also defines course activity as any qualifying course or activity with a clear
purpose and objective which will maintain, improve, or expand the skills and knowledge relevant to a
licensees area of practice. The next rule, 61G15-22.003 F.A.C., sets forth the very specific qualifying
activities for the area of practice, which are:
Successful completion of college courses (one semester hour equals 15 PDH s and one quarter
hour equals 10 PDHs)
Successful completion of continuing-education courses given by FBPE-approved providers (One
contact hour equals one PHD). A list of approved providers can be found on the FBPE web site at
http://www.fbpe.org/pdfs/CEPProviderlist.pdf. The administration of continuing-education
courses can be by any of the following methods:
o Correspondence
o Television
o Internet
o Video Tapes
o Attendance at seminars
o Workshops
o Professional or technical presentations at conventions and meetings

14
Teaching or instructing in any of the above. However, credit can only be taken the first time the
course is taught and credit does not apply to full-time faculty. (One teaching hour equals two
PDHs)
Authoring published papers, articles or books, or accepted licensure exam items for NCEES.
(Equals 10 PDHs)
Patents (Equals 10 PDHs)
Active participation as an officer in professional or technical societies. Credits cannot be claimed
under this provision until the end of the officers term. (One hour of participation equals one PDH
with a two-hour maximum credit for each organization.)
Prior to the 2005-2007 biennium credits taken to satisfy CE requirements in another state could
be counted. However, this rule was repealed.
Prior to the beginning of this biennium, the 2009-2011 biennium, the rules permitted attendance
at a meeting of the Board to be counted as fulfilling the four- hour Law and Rules requirement.
However, that provision was eliminated in late 2008 and Law and Rule credit is no longer
permitted for attendance at Board meetings.

Activities that do NOT qualify are in 61G15-22.005 and are:


Self-generated courses, meaning those courses generated and presented to the licensee himself or
herself;
Personal self-improvement courses
Equipment demonstrations or trade show displays
Enrollment without attendance
Repetitive participation in the same course
Tours of buildings, structures, etc., unless there is a clear objective to build and maintain
competency in a technical field
Regular employment
Personal, estate or financial planning courses
Courses that are below the level of knowledge and skill that reflects the responsibility of an
engineer in charge.

As mentioned earlier, engineers are required to provide a signed statement with the licensure renewal
form and indicate the following:
The title and a description of the activity (college course, seminar, teaching a class, receiving a
patent, etc.)
The date, location and, if provided by an organization, the name of the organization
The area of practice to which the activity applies
The number of PDHs claimed for the activity

In Florida, credits are not allowed to be carried over to the next biennium. All eight hours must be
completed in the biennium preceding the biennium for which renewal is sought.

You should know all providers are required to provide the FBPE with the name and PE number of
engineers completing the Law and Rule and the Area of Practice continuing education requirement. In as
much as the server used to store the continuing education completion information does not reboot to
capture new entries more than once every 24 hours it is likely the report will not be visible and therefore
usable for renewal purposes until the day after it was reported. Although most course providers report
completions on the same day as the course completion providers do have five days to make the
completion report, the point being that it may not be in the best interest of the licensee to wait until that
last day of the renewal cycle to complete continuing education requirements.

15
Be aware also that it is unreasonable to expect to mail in your application on February 28 and receive a
license for the new biennium on March 1. If you plan to sign and seal plans in early March it is in your
interest to submit your application for renewal as early as possible. The FBPE has an aggressive program
to review applications for renewal and, if it is determined that the continuing-education requirement has
not been met, no license will be issued until the requirement has been satisfied. It is the responsibility of
the licensee to maintain sufficient records to demonstrate compliance for at least two licensure periods, or
four years.

There are very limited exemptions from continuing-education requirements and they are:
New licensees who were licensed by examination are exempt for their first renewal period. Under
a proposed rule change not yet final as of the date this Study Guide was written, ALL new
licensees will be exempt from continuing-education requirements for their first renewal period.
Licensees who are in retired status
Licensees in inactive status

The FBPE has set forth substantial requirements of approved providers of continuing education in 61G15-
22.011, 22.012, 22.013 and 22.01. They will not be covered in this document other than to say engineers
should take great care in selecting providers that have been approved by the FBPE. Courses taken from
providers who are not approved will not count for continuing education credit.

Temporary Certificates to Practice


The FBPE may issue a temporary license with the following caveats:
For one specified project in the state
For a period of time not to exceed one year
To an engineer holding a license in another state who is qualified for licensure by endorsement in
a state where Florida-license holders are similarly permitted to work

The FBPE may also issue a temporary certificate of authorization with similar caveats:
For one specified project in the state
For a period of time not to exceed one year
To an out-of-state corporation, partnership or firm provided one of the principals has obtained a
temporary license

Certificates of Authorization
Certificates of Authorization, once called an Engineering Business license, or EB license, are required of
any business entity offering engineering services EXCEPT when the engineering service is offered under
the engineers own given name. As this is commonly misunderstood, a few examples are in order.
Engineering firms called, for example:
John Brown Engineering (legal entity)
John Brown P.E., Incorporated (corporation)
J&B Engineering (fictitious name)
John and Sammy Brown Engineering (partnership)

All of the above names require a Certificate of Authorization. However, if the services are offered or
performed solely by John Brown, P.E., a professional engineer practicing in his given name as it is on his
license, a certificate of authorization is not needed.

If practicing under a certificate of authorization, one or more of the principals of the business or
partnership and all those in the firm who act on behalf of the firm as engineers are required to be licensed.

16
Also, all plans, specifications and other engineering documents filed for record or used by the business
organization shall be dated and bear the seal of the individual who prepared them.

The section on Certificates of Authorization, Chapter 471.023(3),F.S. is very specific that the section does
not relieve the engineer from personal liability for negligence, misconduct or wrongful acts he or she may
have committed and goes into the legalities regarding liabilities for partnerships, business organizations
and shareholders.

Perhaps the most violated section of the entire Engineer Practice Act is Chapter 471.023(4), which
requires holders of certificates of authorizations to notify the FBPE within one month of any change in
the information on the application upon which the certification was based. Updating the certificate of
authorization information is routinely overlooked (as is, by the way, updating the address of record of
professional engineers). As a result, the biennial mailing of renewal applications goes to an incorrect
address and P.E. licenses and certificates of authorization are not renewed on a timely basis and,
subsequently, become delinquent. The Florida Engineers Management Corporation is required to mail
license and certificate renewal information at least 90 days prior to expiration of the license. So, with an
expiration date of February 28th , you can expect to receive a renewal post card reminder for the P.E.
license and the Certificate of Authorization around Thanksgiving or the first of December.

Professional Engineer, Retired


The status of Professional Engineer, Retired or Retired Professional Engineer was placed in the statutes at
the request of engineers who had been lifelong professional engineers and were no longer practicing but
wanted to retain the distinction of the profession. A person must apply for this status and agree to refrain
from the practice of engineering and the use of his or her seal. If P.E. Retired decides he or she wants to
reactivate the license, he or she must make application for licensure and meet the licensure criteria in
effect at the time of application.

Inactive License
Chapter 455.271 F.S. allows each professional board to permit each licensee at the time of renewal to
choose either an active or inactive license. The case for switching to an inactive license is weak from an
economic standpoint. The application fee for the inactive license is $150 and biennial renewal is $75. If
an applicant wants to reactivate the license, the reactivation fee is $150. If the license has been inactive
for more than one year, the applicant must demonstrate completion of 12 hours of engineering-related
education per inactive year including a course in Law and Rules provided by the FBPE. If an applicant
can demonstrate he or she has been in active practice in another state, or has been involved in an aspect of
practice that is exempt, only the Law and Rules course is required.

Delinquent License / Null and Void License


Delinquency of a license is also set out in Chapter 455.271 F.S.(the act that gives the Department of
Business and Professional Regulation regulatory authority), and requires an applicant to affirmatively
reapply with a complete application during the cycle an application becomes delinquent. In other words,
if a licensee fails to renew his license on a timely basis, he or she must submit the renewal application and
the renewal fee of $125 plus a delinquent fee of $100 within the next biennium. Of course, it is a strict
violation to practice engineering with a delinquent license. Failing to renew the license during the
delinquency biennium will cause the license to go null and void on the first day of the second biennium.
In essence, the license will no longer exist and an application for a new license, under the current
licensure requirements, must be initiated.

Certification for Special Inspectors of Threshold Building


Although Florida has avoided the temptation succumbed to by several other states and create a super
license, or a license for structural engineers, Florida does have an additional requirement for engineers

17
who want to offer threshold building inspections. Chapter 471.015(7) directs the FBPE to establish
qualifications for certification of professional engineers as special inspectors of threshold buildings as
defined in Chapter 553.71 and Chapter 553.79 F.S. The concept of having specific qualifications for
threshold buildings (buildings of a certain height, size or occupancy) was first put into law following the
collapse of a condominium during construction, resulting in loss of lives. There was concern that local
governments would each create some sort of qualification procedures, and that there would be great
differences between the various jurisdictions. So Chapter 471.0195 F.S. says P.E.s are not subject to any
standards other than those established by the FBPE.

This section of law is also a little unique in that it requires the FBPE to develop qualifications, not only
for the special inspector of threshold buildings, but for minimum qualifications for the representative of
the special inspector. Traditionally, the engineer is held accountable for the work of sub-professionals in
his employ, and that is still the case for engineers who provide threshold building inspection. However,
because of the nature of threshold building inspection work and the impracticality of the PE / Threshold
Special Inspector being on the job site continuously, the legislature charged the FBPE to develop
minimum qualifications for the Authorized Representative of the Threshold Special Inspector. Those
qualifications are set forth in Chapter 61G15-35 Responsibility Rules of Professional Engineers Providing
Threshold Building Inspections and require at least one of the following:
o licensure as an engineer or an architect
o graduation from an engineering-education program in civil or structural engineering
o graduation from an architectural program
o successful completion of the NCEES fundamentals exam
o Licensure as a building inspector or general contractor.

As mentioned, Florida does not have a super license. However, engineers wanting to offer threshold
building inspector services must submit an application indicating the following qualifications before
being permitted to practice engineering in this area:
o proof of licensure in good standing whose principal practice is structural engineering in
Florida (as Florida does not license engineers by discipline, this may mean passage of the
principles and practice exam in civil, Structural I or Structural II and/or work history in
structural engineering)
o three years experience in performing structural field inspections on threshold buildings
o two years experience in structural design of all structural components of threshold
buildings after having achieved licensure
o experience in structural inspection and/or design of at least three threshold buildings
within 10 years of submitting the threshold inspector application

The Threshold Building Inspector licensure application is available on the FBPE web site at
http://www.fbpe.org/pdfs/apps/SIApp.pdf.

18
CHAPTER 3
The Practice of Engineering

The practice of engineering is all about the sealing, signing and dating of plans, specifications and other
engineering documents. This act of authentication is somewhat unique to the design professions and has
been done in much the same way, with one notable exception, for several generations of engineers. This
section will deal with the how of the engineering practice as it relates to the requirements of the
practice. Subsequent parts of this document will speak to the what (as in what can happen if the
requirements are not met) an engineer can do in his practice of the engineering profession and what he or
she should avoid.

All About Seals


Chapter 471.025 F.S. allows the FBPE to prescribe the form of seal to be used by engineers and requires
those licensed to obtain at least one form of the prescribed seal. For many years the only seal used by
Florida engineers is the embossing impression type seal, the requirements of which are set forth in
Chapter 61G15-23.001 F.A.C. Beginning in November 2009, however, the FBPE changed the rule
regarding seal requirements to allow the use of stamps and Florida engineers have the option of using
either the impression seal or the stamp. In either instance the date and signature must be by the hand of
the engineer and not a stamp.

With the advances in technology, questions have been raised about the use of electronic seals and that
statutes do allow for the use of electronic seal on many legal documents. However, the electronic seal
authorized is NOT a simple scanning of the impression seal, signature and date into a computer to be
copied and pasted into a title block as often as one cares to hit the enter key. One kind of electronic seal
and signature permitted for use by engineers is set forth in a new rule 61G15-23.003 Procedures for
Signing and Sealing Electronically Transmitted Plans, Specifications, Reports or Other Documents. Be
aware that the electronic process is complicated and is not in common use.

As of January 2002, the seal or stamp must be a minimum of 1 7/8 inches in diameter. Previously, the seal
was allowed to be smaller in diameter. However, building officials complained that the small seals were
too cluttered to allow them to be legible. The majority of seal providers today sell a 2-inch seal which, of
course, is acceptable. More recently, the FBPE changed the rule to use the term license within the seal,
instead of the term certificate. Licensees had until January 1, 2006 to obtain a seal with license in the
insert.

When to Seal
61G15-23.002(1) F.A.C. is very specific on when to seal.
It says:
A Professional Engineer shall
o sign by his name and
o affix his seal to
all plans or final drawings
specifications,
reports,
final bid documents provided to the owner or the owners representative, or
other documents prepared or issued by said licensee and being filed for public
record.

19
o date the signed and sealed document

Responsible Charge
61G15-23.002(2) F.A.C. provides even more specifics. It says that each sheet of plans and prints which
must be sealed must be sealed, signed and dated by the engineer in responsible charge. The operative
words in this section of rule are engineer in responsible charge. There is a definition of responsible
charge and it is found in Chapter 61G15-18.011 F.A.C. and says responsible charge relates to
engineering decisions within the purview of the Professional Engineers Act (meaning Chapter 471 F.S.).
Responsible charge does not refer to management control in a hierarchy of professional engineers, except
as each of the individuals in the hierarchy exercises independent engineering judgment and is therefore in
responsible charge. The definition also says responsible charge does not refer to administrative or
personnel management functions, nor does it refer to the concept of financial liability. It means the
engineer who was in engineering control of the work as defined as engineering is the person who must
seal, sign and date the engineering documents.

The definition of Responsible Charge in Chapter 61G15-18.001 F.A.C. is a must read as it dictates the
practice of engineering. To paraphrase, the rule:
Defines Responsible Charge as the degree of control an engineer is required to maintain over
engineering decisions, personally or by others over whom the engineer exercises supervisory
direction and control authority. The engineer in responsible charge is the engineer of record
(engineer of record is defined in Chapter 61G15-30.002).
o The degree of control shall be such that the engineer of record:
Personally makes engineering decisions or reviews proposed decisions when
decisions of public health, safety and welfare are at issue
Be physically present or available in a reasonable time via electronic
communications devices
Judges the validity and applicability of recommendations prior to their
incorporation into the work
o The engineer of record makes decisions pertaining to permanent or temporary work
which could create a danger to the health and safety of the public, such as:
Selection of engineering alternatives and comparison of alternatives
Selection of design standards and materials to be used
Selection or development of methods of testing
Development and control of operating and maintenance procedures
o To determine if an engineer is the engineer of record and in responsible charge, the
following is considered:
Questions concerning engineering decisions should be answered in sufficient
level of detail so there is no doubt as to the engineers proficiency and
involvement in the work.
The answers must demonstrate that the engineer of record made key
decisions and that he or she had the ability to make them
Questions to be answered by the engineer could relate to criteria for
design, applicable codes and standards, methods of analysis, selection of
material and systems, economics of alternative solutions and
environmental considerations
The ability of the engineer to define the span of control and how it was
exercised, and how the engineer was answerable with that span and
degree of control
The engineer must be in charge of and be satisfied with the engineering aspects
of the projects

20
The engineer must have the ability to review design work at any time during the
development of the project and must be available to exercise judgment in
reviewing the documents
The engineer must have personal knowledge of the technical personnel doing the
work and be satisfied that they are capable to perform the work
The engineer approves the inclusion of standard engineering design details, and
conducts reasonable analysis of the content of the standard detail.

In addition to Signing, Sealing and Dating


This same paragraph (2) goes on to say that engineers should, in addition to signing, sealing and dating
each sheet, legibly indicate their name, address and PE number on each sheet. If practicing other than as
an individual under the professional engineers license, the name and address of the business, as well as
the number of the certificate of authorization, should also be on each plan sheet. Fortunately, all of the
information EXCEPT the signature, seal and date can be machine printed in the title block on each sheet
of the plans.

Engineers working for local, state or federal governmental agencies, since they do not have a business
license, should legibly indicate their name and license number, as well as the name and address of their
agency, on each plan sheet.

Specifications and Engineer Reports


Because of the voluminous nature of some specifications, paragraph (2) permits the signing, sealing and
dating of just the cover or index sheet for each section of specifications by the professional engineer in
responsible charge of that section.
Engineering reports are treated similarly and require only the seal, signature and date of the engineer in
responsible charge to be placed on the signature page or cover of the report. In addition to being in
responsible charge of the preparation and production, the engineer signing, sealing and dating
specifications, reports or other engineering documents must have engineering expertise in the engineering
discipline used in the production of the documents.

There will be more on the subject of disciplining errant engineers in a later section, but it does not hurt to
mention more than once that there is a specific reference to sealing, signing and dating engineering
documents in Chapter 471.033 F.S. Disciplinary Proceedings. Paragraph (1)(j) of that section says,
Affixing or permitting to be affixed his or her seal, name, digital signature to any final drawings,
specifications, plans, reports or documents that were not prepared by him or her or under his or her
responsible supervision, direction or control constitutes grounds for which disciplinary action may be
taken.

Certification
Engineers are routinely asked to provide a certification. Chapter 61G15-18.001(4) F.A.C. defines a
certification as a statement that is signed and/or sealed by an engineer representing that the engineering
work addressed in the certification was performed by the professional engineer, is based on his
knowledge, information and belief, and is in accordance with commonly accepted procedures consistent
with applicable standards of practice. A certification is not, by definition, a guarantee or warranty.

Chapter 61G15-29 F.A.C. also speaks to certifications and suggests that when an engineer is presented
with a certification, it should be carefully evaluated to determine if any of the following circumstances
apply. If they do, the engineer should modify the certification to limit the scope of services actually
provided, or decline to sign and seal it. Those red flags for problem certifications are:
relate to matters beyond the engineers technical competence
relate to matters beyond the scope of services provided

21
relate to matters not prepared under the engineers supervision, direction or control

Successor Engineer Adopting the Work of Another Engineer


Few procedures cause more consternation to engineers than how to reuse the work of another engineer.
There are three important points to remember when seeking to reuse already sealed documents of another
engineer. They are: documentation, documentation and documentation. Rules 61G15-27.001 F.A.C. sets
the criteria for that required documentation. If requested, the successor engineer must be able to:
Produce evidence that the engineer has recreated the work of the original engineer including
o calculations
o site visits
o research
Plans do not need to be redrawn. However,
o the successor engineer must be able to document his or her effort of re-thinking and re-
working the entire design process, and
o the engineer must remove the original engineers title block from the plans, and
o insert his or her own title block, and
o sign, seal and date the document.
In doing these things, the successor engineer takes complete and total responsibility for the
documents and is fully liable for the work.

Section (2) of this same rule sets forth the final step in adopting the work and requires, before the work is
signed, sealed and dated, the successor to notify the original engineer, his successors or assigns by
certified letter of his intentions to reuse the original work and to take full responsibility as though it was
the successors original work.

Responsibility Rules
In 1993, the FBPE began adopting a series of rules called responsibility rules to promote
Proper conduct in the practice of engineering
Due care
Regard for acceptable engineering principles and standards
It was an interesting concept at the time as the purpose of the rule, as stated in 61G15-30.001 F.A.C.,
indicated that the engineer may avoid disciplinary actions if he or she adhered to the various
responsibility rules. Otherwise, the engineer may be in non-compliance with rule 61G15-19.001(4),
F.A.C., which requires an engineer to not be negligent in his or her practice. There are several outs to
the responsibility rules also stated in the Purpose paragraph. The responsibility rules do not have to be
followed if:
Deviation from the rule is justified under specific circumstances of the project and is the
sound professional judgment of the engineer, or
When contractual relationships do exist as long as the contractual relationships do not
violate Chapter 471 F.S. or the intent of the responsibility rules of the FBPE.

The Responsibility Rules have been very helpful in promoting proper conduct in the practice of
engineering, including the Definitions Common to All Engineers Responsibility Rules (61G15-30.002).
Prior to adoption of the definitions of such key terms as Engineer of Record, Prime Professional,
Delegated Engineer and Public Record, the understanding of these terms varied by the contract
documents in use and there was little commonality. The adoption of the rule defining key engineering
terms put everyone on the same proverbial page and allowed for better understanding of engineering

22
responsibilities, hence promoting proper conduct in the practice of engineering and due care and regard
for acceptable engineering standards. Terms defined in rule 61G15-30.002 F.A.C. include:
Engineer of Record
A Florida PE who is in responsible charge for preparation, signing, dating, sealing and issuing
engineering documents for any engineering service or creative work.
Prime Professional
A Florida PE or qualified engineering corporation or partnership who is engaged by a client to
provide planning, design, coordination, arrangement and permitting for the project and for
construction observation in connection with the engineering project, service or creative work.
The Prime Professional may also be the Engineer of Record on the same project.
Delegated Engineer
A Florida PE who undertakes specialty service and provides services or creative work
regarding a portion of an engineering project. A Delegated Engineer usually falls into one of the
following categories:
o an independent consultant
o an employee or officer of an entity supplying components to a fabricator or contractor, as
long as the engineer acts as an independent consultant or through a qualified corporation
or partnership
o an employee or officer of a fabricator or contractor, so long as the engineer acts as an
independent consultant or through a duly qualified corporation or partnership.
Engineering Document
Engineering Documents are designs, plans, specifications, drawings, prints, reports or similar
instruments of service in connection with engineering services or creative work that have been
prepared and issued by a PE or under his or her responsible supervision, direction and control.
Delegated Engineering Document
Delegated Engineering Documents are those engineering documents that are prepared by a
delegated engineer.
Public Record
An engineering document is said to be filed for public record when it is presented with the
engineer of records knowledge and consent to any federal, state, county, district,
authority, municipal or other government agency in connection with the transaction of official
business with the agency.
Engineering Documents Filed for Public Record
This is a new definition and means those documents filed for public record with the Authority
having Jurisdiction to determine compliance with Codes and Standards and to be used in the
execution of the project.
Shop Drawings
These are defined for the first time as drawings depicting installation means and methods, catalog
information on standard products, prepared by a contractor, manufacturers or professional
engineers, for incorporation into the project which are prepared based on an engineering direction
contained in the Engineering Documents. Shop Drawings do not require the signature, date and
seal of a professional engineer.

Record Documents
These are the documents that are a compiled representation of the constructed project. If the
engineer is relying on information provided by others not under his supervision and control, the
engineer is not required to sign, date and seal these documents. If the engineer is relying on
information by others, the following should be included in the documents:
o A statement that the documents are a compiled representation of the constructed project

23
o A list of sources and the basis of information used in preparation of the documents
o A statement that the documents are believed to be correct

Minimum Requirements for Engineering Documents


o When prepared for inclusion with an application for a building permit, engineering
documents shall meet all Engineer Responsibility Rules and be of sufficient clarity to
indicate the location, nature and extent of the work proposed and show in detail that it
will conform with provisions of the Florida Building Code and relevant laws and shall
include:
Information that provides material specifications required for the safe operation
of the system that is a result of engineering calculations, knowledge and
experience
List the federal, state, municipal and county standards, code and rules with their
effective dates with which the engineering documents are intended to conform
Information as determined by the engineer of record needed for the safe and
efficient operation of the system
List of engineering-design criteria, reference project specific studies, reports and
delegated engineering documents
Identify elements of the design that vary from governing standards and identify
the alternative method used to ensure compliance with the Responsibility Rules
o Engineers are required to legibly state their name and business address on engineering
documents, and documents issued for preliminary or conceptual use need to have that
clearly stated on the document.
o When an engineer does not intend to accept responsibility for certain elements on the
document, the engineer must note either the exception of his responsibility or, in the
alternative, the extent of his responsibility, on the document.
o Drawings shall be legible and clearly define delineated work. Drawings must also comply
with rule 61G15-23, F.A.C., relating to seals.
o Preliminary documents must indicate the documents are not in final form but are being
transmitted to the agency to receive agency review and comments. Preliminary
documents may subsequently be revised to reflect resolution of issues. These changes or
revisions may prompt additional document submittal for agency approval on the same
project.

Delegation of Engineering Documents: Obligations of the Engineer of Record


An engineer of record (a P.E. in responsible charge for preparation, signing, sealing and dating
engineering documents) who delegates a portion of his work to a delegated engineer (a P.E. who
undertakes a specialty service and is the engineer of record for that particular service) is obligated
to communicate in writing the engineering requirements to the delegated engineer. The engineer
of record is then required to review the work of the delegated engineer for compliance and
confirm that:
o The delegated work was prepared by an engineer
o The delegated work conforms with the intent of the engineer of record and meets the
written criteria
o The effect of the delegated engineers work on the overall project conforms with the
intent of the engineer of record.

Delegation of Engineering Documents: Obligations of the Delegated Engineer of Record


The delegated engineers responsibility is to review the engineer of records written requirements
to determine the scope of engineering and, of course, comply with the written requirements
received. The delegated engineers documents must include the project identification and

24
preparation criteria. If there are details or features that conflict with the written requirements
provided by the engineer of record, the delegated engineer must contact the engineer of record to
resolves the conflict. Of course, the delegated engineer must send the delegated engineering
documents to the engineer of record for review and all final documents must be signed, sealed
and dated by the delegated engineer and include:
o Drawings introducing engineering input
o Calculations
o Computer printouts may be substituted for calculations provided they are accompanied
by sufficient assumptions and input /output information to permit evaluation.

Prime Professional Responsibility


This one is simple. If there is a prime professional, it is the responsibility of the prime
professional engineer to retain and coordinate the services of such other professionals as needed
to complete the project. A prime professional can also be the engineer of record for the project or
portion of the project.

Use of Computer Software and Hardware


This one is simple also. The engineer is responsible for the results generated by any computer
software or hardware he or she uses in providing engineering services.

Retention of Engineering Documents


At least one copy of all signed, sealed and dated documents and all calculations shall retained by
the licensee or the licensees employer in a readily accessible format for a minimum of three
years from the date they were sealed.

Energy Conservation Compliance


The engineer who prepares compliance calculations and certifies accuracy must verify that the
building construction documents conform to the compliance calculations. The data used in the
calculations are required to be signed, sealed and dated.

Specific Responsibility Rules


Following the Responsibility Rules Common to All Engineers, there are a series of rules specific to
certain applications of engineering. In other words, in addition to rules promulgated to promote
proper conduct in the practice of engineering and due care and regard for acceptable engineering
principles and standards for all engineering, the FBPE adopted responsibility rules particular to
certain areas of practice. These specific responsibility rules arespecific, in that they contain
extremely detailed information unique to these particular areas of practice. It is strongly
recommended that new licensees review the responsibility rules pertinent to their area of practice
against their office practices in order to avoid disciplinary action as failure to comply with the rules
may be considered negligence.

Design of Structures
Within rule 61G15-31, F.A.C., the Design of Structures there are specific standards.
Currently, the following standards are in place:
o Design of Structures Utilizing Prefabricated Wood Trusses
o Design of Cast-In-Place Post-tension Concrete Structural Systems
o Design of Structures Utilizing Pre-cast and Pre-stressed Concrete
o Design of Structural Systems Utilizing Open Web Steel Joist and Joist Girders
o Design of Pre-engineered Structures
o Design of Foundations

25
o Design of Structural Steel Systems

Design of Fire Protection Systems


Similar to the Structures rule, the rule for Fire Protection Systems, 61G15-32, F.A.C.,
provides a general statement pertaining to the responsibility of engineers involved in this type
design, as well as a number of definitions pertinent only to the following kinds of fire
protection systems:
o Design of Water Based Fire Protection Systems
o Design of Gas Agent Fire Suppression Systems
o Design of Foam and Foam Water Fire Suppression Systems
o Design of Dry Chemicals and Miscellaneous Fire Suppression or Control Systems
o Design of Fire Alarms and Detection Systems (This rule section was substantially re-
written in 2008. See Chapter 6 for new language).
o Design of Fine Water Spray (Mist) Fire Suppression and Control Systems

Design of Electrical Systems


As in the Structures and the Fire Protection Responsibility Rules there is a particular set of
standards for the Electrical Systems (61G15-33, F.A.C.) following a statement of general
responsibility for the electrical engineer as well as several definitions.
Current responsibilities for the following are set for:
o Design of Power Systems
o Design of Lighting Systems
o Design of Communications Systems
o Design of Alarm and Signaling Systems
o Design of Lightning Protection Systems
o Design of Grounding Systems
o Design of Instrumentation and Control Systems
o Certification of Electrical Systems of Public Interest

Design of Mechanical Systems


As in the other disciplines there a Responsibility Rule for Mechanical Systems (61G15-34,
F.A.C.) is a statement of general responsibility, a definition and enumeration of specific
responsibilities. The specific requirements are spelled out in:
o Design of Heating, Ventilation and Air Conditioning Systems
o Design of Process and Fluid Flow Systems
o Design of Heat and Energy Transfer Systems
o Design of Material and Human Transfer Systems
o Design of Plumbing Systems
o Design of Mechanical Machines and Motions Systems
o Design of Instrumentation and Control Systems

Engineers Providing Threshold Building Inspections


The Responsibility Rule for Professional Engineers Providing Threshold Building Inspections
(61G15-35, F.A.C.) was put into the responsibility rules in 2001 pursuant to Section 553.79 F.S.
for those engineers wanting to provide threshold building inspections. It provides definitions and
qualifications for threshold building inspectors, as well as common requirements. The Common
Requirements for Engineers Providing Threshold Building Inspections Services as Special
Inspectors requires:

26
o Certification that the Special Inspector is competent to provide engineering services for
the specific type of structure
o Assurance that any Authorized Representative is qualified by education and licensure to
perform the duties assigned by the Special Inspector
o The Special Inspector be in Responsible Charge of the work of the Authorized
Representative
o Special Inspectors to institute certain quality assurance procedures

Product Evaluation
The Responsibility Rule for Professional Engineers Providing Product Evaluation (61G15-36,
F.A.C.) defines procedures, materials, devices, methods of construction, and installation of a
product or group of products and provides related definitions and common requirements to all
product evaluation documents.

27
CHAPTER 4
Acts for Which Engineers May be Disciplined

Penalties the FBPE May Impose


The crux of the Engineer License Act dealing with discipline is found in Chapter 471.033 F.S., and it
states its thrust in a very appropriate way. It clearly lists the acts for which an engineer can be disciplined
and, in the same section, points to what punishment the FBPE can impose. For emphasis, we will list the
penalties that await engineers should they be found guilty of violating any number of acts meriting
disciplinary action. Disciplinary action may include:
Revocation of a license
Suspension of a license
Fine an engineer up to $5,000 for each count or offense
Place an engineer on probation for any period of time plus subject the engineer to such conditions
of practice the FBPE may specify
Restrict the scope of practice of an engineer
Require restitution
Reprimand an engineer
Deny an application for licensure

Add to this the requirement that the FBPE has to publish the names and the offenses of the engineer, and
it is apparent that any act leading to disciplinary action should be avoided.

Acts to Avoid
Those acts to avoid are listed in Chapter 471.033 (1) F.S. which states in paragraph (2) that the FBPE will
specify by rule what acts or omissions constitute a violation of paragraph (1). Those acts are:
Violation of:
o Chapter 471.025 F.S. dealing with signing, sealing and dating engineering documents
o Chapter 471.031 F.S. which states that a person (not necessarily an engineer, but a
person) may not, without committing a misdemeanor of the first degree:
Practice engineering unless licensed or exempt from licensure
Use the title professional engineer or other title tending to indicate licensure
(certain persons who are exempt from licensure may use the title engineer as
long as it does nor connote licensure)
Present as his or her own the license of another
Give false or forged evidence to the FBPE
Use a license that that has been suspended, revoked or is inactive or delinquent
Employ nonexempt unlicensed person to practice engineering
Conceal information relative to this chapter
Or any of the following
o Attempting to get a license by bribery or fraudulent misrepresentation
o Having a license acted against by another licensing authority for any act that would be a
violation of Chapter 471 or Chapter 455 F.S.
o Being convicted or found guilty of, or entering a plea of nolo contendere to, regardless of
adjudication, a crime in any jurisdiction which relates directly to the practice of
engineering

28
o Making or filing a report (meaning a report requiring the signature and seal of an
engineer) the licensee knows to be false, willfully failing to file a report required by law,
willfully impeding such a filing or inducing another to impede the filing
o Advertising goods or services in a fraudulent, false or deceptive form
o Engaging in fraud, deceit, negligence, incompetence or misconduct in the practice of
engineering
o Violating Chapter 455 F.S.
o Practicing on a revoked, suspended, inactive or delinquent license
o Affixing his or her seal, name or electronic signature to final engineering documents not
prepared by him or her or under his or her responsible supervision, direction or control
o Violating any order of the FBPE or Department previously entered in a disciplinary
hearing

Grounds for Disciplinary Proceedings


Some of the acts listed above are a clear violation of the engineer Practice Act on their face. However,
several of the acts are clarified in rule 61G15-19.001 F.A.C. Grounds for Disciplinary Proceedings.
Those acts further explained are:
Advertising
The engineer shall not advertise in a false, deceptive or misleading manner means the
statement is misleading or deceptive if:
o It contains a material misstatement of fact
o Omits to state any material fact necessary to not make the statement misleading
o Is likely to create an unjustified expectation
o Implies the engineer is a specialist outside of his area of expertise
o It contains a representation of implication that is likely to cause an ordinary person to
misunderstand or be deceived, or fails to contain reasonable warnings or disclaimers to
make the representation not deceptive
o Falsifies or misrepresents the extent of education, training or experience implying
qualification for selection of employment or engagement, including misrepresentation or
exaggeration of prior assignments
o Misrepresentation of pertinent facts concerning the engineer employer, employees,
associates, or past accomplishments for the purpose of enhancing his qualifications and
his work
Corporate or Fictitious Names
When practicing under an assumed, fictitious or corporate name, the name cannot be misleading
as to identity or the responsibility of those practicing under it, or it is otherwise fraudulent,
misleading and deceptive as described above. In addition:
o An engineer must have a certificate of authorization when practicing under any other
name than his own given name as it is on his license
o A corporate name may include the names of deceased or retired members or of a
predecessor firm in a line of succession
o An engineering firm may not offer services under a firm name which contains only the
name of a person who is not an engineer unless the person is licensed as an architect, land
surveyor, landscape architect or professional geologist
Negligence
Negligence means failure of the engineer to utilize due care in the performance in an engineering
capacity, or failing to have due regard for acceptable standards of engineering principles. Failure
to follow the procedures of the Responsibility Rules is considered negligence unless deviations or
departures from the rules are justified.
Incompetence

29
Incompetence means the physical or mental incapacity or inability of an engineer to be able to
perform the duties normally required of an engineer.

Misconduct
Misconduct means a number of acts and is the most cited violation by the FBPE.
Acts of misconduct include:
o Expressing a public opinion on an engineering subject without being informed as to the
facts
o Being untruthful, deceptive or misleading in a professional report or statement, or
omitting relevant or pertinent information in a statement, the result of which omission
would or reasonably could lead to a fallacious conclusion on the part of the client or
employer
o Performing work when not qualified by training or experience in the practice involved
o Signing and sealing work in a subject matter over which the engineer lacks competence
because of inadequate training or experience
o Offering a bribe, gift or commission to get selection or preference for employment,
except for payment of the usual commission for securing salaried positions through
licensed employment agencies
o Conflict of interest with an employer or client without knowledge and approval of the
client or employer, unless the conflict is unavoidable and the engineer does the following:
Discloses in writing to the client or employer the possible conflict
Assures in writing the conflict will not influence the judgment of the engineer or
the quality of services
Promptly informs his client in writing of any association or interest that may
influence his judgment or quality of service
o Soliciting or accepting valuable considerations from material or equipment suppliers for
specifying their products without the written consent of the employer or client
o Use of engineering expertise or engineering status in commission of a felony
o Signing and sealing engineering documents that were not done by the engineer or done
under his supervision and control
o Allowing the use of his name or firm name in a business he has reason to believe is
engaging in business practices of a fraudulent nature
o Failure to inform his employer or responsible parties when his engineering judgment is
overruled by an unqualified authority and there is possible threat of danger to the health
and safety of the public
o Failure to notify the FBPE when the engineer has knowledge or reason to believe any
person or firm has violated the Engineer Practice Act or the rules of the board (Chapter
471 F.S. or Chapter 61G15 F.A.C.)
o Violation of any Florida law directly regulating the engineer practice
o Failure to obey the terms of a final order by the FBPE imposing discipline
o Making statements, criticism or argument of an engineering matter which is paid for by
an interested party, unless the engineer identifies the interested party on whose behalf he
is speaking and the interest he or the party has in the matter
o Signing and sealing all documents for an entire engineering project unless each design
segment is signed and sealed by the engineer in responsible charge of preparing the
design segment
o Revealing facts, data or information gained in a professional capacity without prior
consent of the client or employer

Professional Engineer as a Building Code Official

30
Chapter 471.045 F.S. allows a licensed engineer to provide building code inspection services
upon the request of a local government or state agency without being certified by the Florida
Building Code Administrators and Inspectors Board. An engineer cannot provide plans review as
an employee of a local government upon any job that the engineer or his or her company
designed. In the capacity as a building code inspector or plans examiner, the engineer is subject to
discipline for:
o Violating or failing to comply with any provision of Chapter 471 or rules of the FBPE
o Having been convicted of a crime which directly relates to building code inspection or
plans examination
o Filing a false report or record
o Inducing another to file a false report or record
o Failing to file a required report or record
o Impeding or obstructing such a report or record
o Inducing another to impede or obstruct filing a report or record

Penalties That May Be Imposed


Chapter 61G15-19.004 sets forth a range of penalties that the FBPE may impose on engineers guilty of
violating Chapter 471 F.S. The purpose of the guidelines is to give engineers notice of the range of
penalties which can be imposed. The guidelines are based on a single count of violation and multiple
counts or other violations in the same complaint will be grounds to enhance the penalties. The guidelines
are lengthy, so only representative examples of the most frequent violations will be provided. They are:
Failure to date plans
Minimum: Reprimand
Maximum: Reprimand plus one (1) year probation
Plan Stamping (signing and sealing work not prepared by the engineer)
Minimum: Reprimand, plus one (1) year probation, plus $1,000
fine
Maximum: Reprimand, plus one (1) year suspension, plus two
years probation, plus $5,000 fine
Violation of a Final Order
Minimum: Suspension plus $1,000
Maximum: Revocation plus $5,000 fine
License disciplined by another jurisdiction
The same penalty as imposed in the other jurisdiction or as close
as possible to the penalties in the Florida law
Negligence
Minimum: Reprimand, plus two (2) year probation, plus $1,000
fine
Maximum: Reprimand, plus five (5) year suspension, plus ten
(10) year probation plus $5,000 fine

Aggravating or Mitigating Circumstances


The FBPE is permitted to deviate from the guidelines upon a showing of clear and convincing
evidence prior to imposing penalty of any aggregating or mitigating circumstances.
Aggravating circumstances that may allow a penalty greater than the maximum in the guidelines
include:
o History of previous violations
o In cases of negligence, the magnitude of the project and the damage inflicted upon the
public
o Violations of practice acts in other jurisdictions

31
o Violations of the provision of the practice act where a letter of guidance had been
previously issued
Mitigating Circumstances which may justify penalties less than those in the guidelines include:
o In cases of negligence, the minor nature of the project and lack of danger to the public
o Lack of previous disciplinary history
o Restitution of damages suffered by the licensees client
o The licensees professional standing, including his continuing education history
o Steps taken by the licensee or his firm to insure non-occurrence of similar violations in
the future
o
The rules of the FBPE also provide for a penalty process other than the standard administrative
complaint process for first time, lesser infractions. These alternatives include:
Notice of Noncompliance
A Notice of Noncompliance can be issued for an initial offense in the following violations:
o Failure to date documents when signing and sealing
o Practice with a delinquent or inactive license for less than one month
o Practice with a delinquent certificate of authorization for less than one month
o Failing to report a criminal conviction or plea of nolo contendere, regardless of
adjudication, pursuant to Section 455.227(1)(t), F.S., if the conviction or plea occurred
prior to July 1, 2009. This subparagraph shall remain in effect until July 1, 2012.
Citations
For second offenses of the type covered by a Notice of Noncompliance, and for the following, the
FBPE may also issue citations and impose fines. The offenses covered by citations and their
fines are specified in Chapter 61G16-19.0071 F.A.C. and are:
o For practicing with an inactive or delinquent license for more than one month, the fine is
$100 per month or fraction thereof
o For failure to notify the FBPE of a change in principal officer that is the qualifying
officer for the corporation or partnership within one month, the fine is $500
Before the citation can be issued, the investigator must confirm the violation has been corrected
or is in the process of being corrected. If the engineer does not dispute the matter in the citation
within 30 days, the citation becomes the final order of the FBPE and becomes a matter of public
record.

32
CHAPTER 5
The Legal Process

This last section is a brief over view of the legal process that involves the filing of a complaint, the
subsequent investigation and the prosecutorial procedures that lead to a Final Order or judgment by the
FBPE. Although not an integral part to the licensing and practice of engineering we are including this
brief overview of the legal process as information to future licensees should a complaint be filed against
them. The legal process is not complicated, but strong advice should be offered to any engineer against
whom a complaint is filed. The incontrovertible advice is to hire an attorney.

It is true the legal process is not complex, but the filing of a complaint places in jeopardy a profession that
requires four or more years of rigorous academic preparation, four additional years of active experience,
two very tough eight-hour examinations and untold sacrifices by the engineer and his family. Hiring a
good lawyer is a good first step. Hiring a lawyer who is qualified in the area of administrative law is even
better, and hiring a lawyer experienced in administrative law and with first-hand knowledge of the FBPE
and a working relationship with the staff is the best course of action.

Chapter 455.225 F.S. sets forth the disciplinary proceedings that will be followed by the FBPE and
include:
The Filing of a Complaint
o The FBPE will investigate any complaint if the complaint is
in writing and
signed by the complainant and
legally sufficient
o The FBPE will investigate an anonymous complaint if
it is legally sufficient
it is a substantial violation and
if initial review leads the FBPE to believe the allegation is true
o Legally Sufficient means the complaint contains facts that, if true, violate any practice
act or rule of the FBPE.
o When an investigation is initiated, the FBPE must provide the subject of the complaint
with a copy of the complaint.
o The subject has 20 days to provide a written response to the complaint.
o The Probable Cause Panel (similar to a Grand Jury) of the FBPE must consider the
subjects response when reviewing the complaint.

The Investigative File


The investigative file contains the complaint, the subjects response to the complaint and the
findings of the investigation. Upon completion of the investigation, and at the written request of
the subject and at his expense, the file will be sent to the subject. The subject has 20 days to file a
written response to the information in the file.

The Probable Cause Panel


The Probable Cause Panel is comprised of three members of the FBPE or two members of the
FBPE and a former member of the FBPE and they are recused from sitting in judgment on cases

33
they reviewed while serving on the Probable Cause Panel. Upon receipt of the investigative file,
and the recommendation of prosecuting attorney regarding the existence of probable cause, the
Probable Cause Panel determines probable cause.

A finding of probable cause


o If no probable cause is found, the information becomes public and the case is dismissed.
o In lieu of probable cause, the Panel may issue a letter of guidance to the subject. Per
471.038(7), and unlike other regulatory boards the file becomes public information in 10
days whether or not probable cause is found
o If probable cause is determined to exist, the information becomes public in 10 days and
the panel issues an administrative complaint.

Options if probable cause found


o If there is a dispute of facts, a formal hearing can be held before an Administrative Law
Judge. The Administrative Law Judge will issue a finding as to the facts and a
recommended Final Order to the FBPE. The FBPE may or may not accept the
recommendation of the Administrative Law Judge before issuing the Final Order.
o If there is no disagreement as to the facts, an informal hearing may be requested before
the FBPE to either argue interpretations of the law or offer mitigating circumstances.
There are substantial risks when throwing yourself on the mercy of the court, so to
speak, and this option should be considered very carefully. Please remember that the
argument everyone does this and I only did this because my client asked me to are
not persuasive and will not be received well by the FBPE.
o The subject can agree to a settlement stipulation, or as they say on the television show,
Law and Order, cop a plea. Perhaps the option most used as it allows for calculated
negotiation between experienced and professional counsel and removes problems
associated with emotionally charged statements uttered on the record in front of
professional peers set to pass judgment.
o Do nothing is the last and least preferred option. This is the option of the default
judgment and penalties have been known to be significant in the absence of any reason
for violation of the law.

In Summary
This section is not intended to prepare you for the section of the Florida Bar Examination of Procedures,
but to provide a brief overview of the sequence of the complaint process and the options of the engineer
should he find himself the subject of a complaint. If nothing else is learned from this section but get a
lawyer, the space is well served.

34
QUESTIONS
Chapter One

1. Florida was the first state to enact an all-inclusive Engineer Practice Act
a. True
b. False

2. After receiving a complaint, FEMC has ____ days to determine if it is


legally sufficient.
a. 15
b 30
c 90
d. 30 days after the Probable Cause Committee meets

3. Key definitions important to Engineers are found in:


a. 61G15 F.A.C. (The Rules of the Board)
b. Chapter 471 Florida Statutes
c. The Responsibility Rules
d. All of the above

Chapter Two

4. Engineer Brown has been engaged in providing consulting engineering


services for a number of years as a sole proprietor. Because his practice
was growing, he recently hired another engineer and decided to
commence offering professional services under the name of Brown and
Associates. In order to comply with the rules he must:
a. obtain a new engineering seal
b notify the Board office of the name change
c. obtain a Certificate of Authorization
d. no action by Engineer Brown is necessary

5. Unless otherwise exempt, in order to do the activities described in


471.005(7), F.S., a person must:
a be a graduate of an approved engineering curriculum
b have four years of acceptable engineering experience
c have passed the Principles and Practice examination
d be licensed

6. The incidental practice exemption in Chapter 471 involves work


performed by:
a. civil or structural engineers whose principle practice is civil or
structural engineering
b. architects
c. both a. and b.
d engineering faculty

35
7. Which of the following activities are not exempt from licensure pursuant to
Chapter 471, F.S.?
a. A person practicing engineering on property owned by him or her
b. A full time electrical engineer of Progress Energy Corporation
c. A civil engineer employed full time by the U. S. Army Corps of
Engineers
d. An independent consultant working on the design of an electrical
distribution system project for Progress Energy Corporation.

8. Applicants for licensure with degrees from foreign institutions are required
to document substantial equivalency to ABET criteria to the FBPE. They
can do this by:
a. providing a transcript from their institution to the Board
b. providing a notarized certification that they have completed the
requisite college credit hours set forth in Chapter 61G15-
20.007(2)(a) thru (2)(d)
c. getting the evaluation of substantial equivalency from a provider of
the service that is approved by the FBPE
d. passing the Principles and Practice examination

9. In order to verify an applicants experience, the FBPE


a. follows guidelines set forth in Chapter 61G15-20.002
b. relies on information obtained from personal references
c. requires evidence of employment from employers or supervisors
who are employed in the engineering profession
d. all of the above

10. A person employed by a post secondary educational institution and


teaching an engineering design course is required to be licensed.
a. True
b. False

11. The criteria that must be met by a foreign student to document substantial
equivalency to A.B.E.T. criteria is:
a. determined by the institution which granted the degree to the
foreign student
b. found in relevant sections of Chapter 61G15 (The Rules of the
Board)
c. determined by the F.B.P.E. approved provider evaluating the
application
d. found in Chapter 471

12. Continuing education credits earned in excess of the eight required for
license renewal may be carried over to the next biennium.
a. True

36
b. False
13. Persons exempt from licensure as an Engineer are:
a. listed in Chapter 471.003 F.S.
b. Architects performing engineering services incidental to their
practice
c. Engineering faculty
d. All of the above

14. If an applicant for licensure fails the Fundamentals or Principles and


Practice Examination ______times the FBPE will require the applicant to
take ______
Hours of additional college level courses with grades no lower than a C
as a condition of future eligibility to retake the examination.
a. three, 15
b. five, 12
c. three, 12
d. This provision is only applicable to foreign degree applicants

15. A licensed building inspector or general contractor may be an authorized


representative of the Threshold Building Inspector.
a. True
b. False

16. Upon initial licensure by exam in Florida, you will be required to pay the
renewal fee during the first renewal period, but you are exempt from
obtaining continuing education credits.
a. True
b False

Chapter Three

17. The Chief Financial Officer (CFO) of an engineering corporation, who is a


P.E. noted that one of their projects was extremely over-budget. The
CFO, in a memo to the Project Manager, stated that necessary steps be
taken to complete the project in a timely manner and bring it back within
budgetary limitations. As a result of the memo:
a. the corporation assumed responsible charge for the project
b. the CFO became the engineer in responsible charge
c. there was no change in the person in responsible charge
d. responsible charge was shared equally by the CFO and the project
manager

18. A client becomes dissatisfied with the progress that Engineer A is making
on his Project. As a result, he terminates Engineer A and hires Engineer
B to complete the work. Engineer B:

37
a. must be able to document his or her effort of reworking the entire
design process
b. must take complete responsibility for the documents
c. must notify Engineer A, by certified mail, of his intentions to reuse
already sealed documents
d. all of the above

19. In order to promote proper conduct in the practice of engineering, due care,
and regard for acceptable engineering principles and standards, the FBPE
adopted:
a. a rule setting forth minimum requirements for engineering
documents
b. a rule defining engineering terms
c. rules specific to certain applications of engineering disciplines
d. responsibility rules

20. The engineer in responsible charge is:


a. the project manager
b the engineer of record
c. the principal engineer on a project
d. the project administrator

21. The engineer of record may exercise control over a project by means of
electronic communication devices,
a. True
b. False

22. A Florida PE, who is an independent consultant, prepares the design and
drawings for prefabricated wood trusses for the roof of a structure
pursuant to the engineering requirements prescribe by the engineer of
record. The P.E. for the roof trusses is known as:
a. a subconsultant
b. a specialty engineer
c. a delegated engineer
d, none of the above

23. The form of seal permitted to be used by Florida PEs is:


a. the embossing impression type
b. a stamp
c. an electronic seal
d. any of the above

24. Certifications, when presented to an Engineer, should be carefully


evaluated. The engineer may:
a. sign them
b. modify them

38
c. decline to sign them
d. any of the above

25. The Responsibility Rules do not have to be followed if:


a. The project is for a federal, state or local government agency
b. The project has a value of $100,000 or less
c. Deviation from the rule is justified under special circumstances of
the project and is the sound professional judgment of the Engineer
d. he project is being executed under a design-build contract

26. The data used in the calculations to certify energy conservation


compliance must be signed, sealed and dated.
a. True
b. False

27. The minimum requirements for engineering documents are enumerated in:
a. The Florida Building Code
b. Chapter 471, F.S.
c. Engineer Responsibility Rules.
d. Rule 61G15-23 F.A.C. relating to Seals

Chapter Four

28. Engineer Smith, who is licensed in several jurisdictions, recently had his
license in one of the jurisdictions acted upon by the licensing authority in
that jurisdiction. Engineer Smith should expect the following to occur in
Florida:
a. Nothing, since the infraction did not happen in Florida
b Assuming the infraction in the other jurisdiction is an infraction in
Florida, having approximately the same penalty imposed by the
FBPE as was imposed in the other jurisdiction
c Eight (8) hours of continuing education courses will be required to
maintain the Florida license
d. None of the above

29. While rules of the Board set forth a range of penalties the FBPE may
impose on engineers for law and/or rule infractions, the Board is permitted
to deviate from the guidelines due to:
a. aggravating circumstances
b. mitigating circumstances
c. for first time, lesser infractions, issuance of a notice of
noncompliance or citation
d. all of the above

39
30. Failure to have due regard for acceptable standards of engineering
principles is considered:
a. Negligence
b. Misconduct
c. Incompetence
d. Performing work when not qualified by training or experience

Chapter Five

31. If probable cause has been found against a licensee, the least preferred
settlement option is:
a. request a formal hearing before an Administrative Law Judge
b. request an informal hearing before the FBPE
c. agree to a settlement stipulation
d. do nothing and accept a default judgment
.
32. The FBPE will investigate any complaint if it is:
a. in writing
b signed by the complainant
c. legally sufficient
d. all of the above

33. An Engineer against whom a complaint is filed should:


a. determine if the complaint, in his or her opinion, is legally sufficient
b. provide a written response to the complaint within 20 days
c. let the complaint process run its course
d. assemble data related to the subject of the complaint

40

Вам также может понравиться